GMAT Critical Reasoning Discussions

Where can i get my hands on the powerscore CR bible in delhi?


Thanks
@snaval1 sorry for late reply...its B .

Hi all, I am bit confused with the CR question I read in a forum
The question is

"A recent study has found that aerobic exercising is the best bet to reduce fat compared to resistive training. A research center X had done a head-to-head comparison on aerobic training, resistive training, combo of both and found that
aerobic exercising is the most efficient and effective way to reduce one's belly fat. Aerobic exercise reduce 70% more calories compared to resistance training.

Which of the following if true, weakens the argument?

A. Pilates training, if done for regularly for 6 months can be more effective than the aerobic training

B. Aerobic exercise, if done without proper training can lead to injuries in neck and back bone.

C. 70 % of the people who reduce fat by aerobic training have gained weight in the next 6 months

D. Aerobic training if not combined with resistive training can reduce the muscle's strength.

E. Research center X's study has not been verified independently. Although, the sample were from diverse communities, the total number of people were not enough to draw a conclusion
"


The answer is A. I'm confused why not E. What if the research center study is flawed.

Hi all, I am bit confused with the CR question I read in a forum
The question is

"A recent study has found that aerobic exercising is the best bet to reduce fat compared to resistive training. A research center X had done a head-to-head comparison on aerobic training, resistive training, combo of both and found that
aerobic exercising is the most efficient and effective way to reduce one's belly fat. Aerobic exercise reduce 70% more calories compared to resistance training.

Which of the following if true, weakens the argument?

A. Pilates training, if done for regularly for 6 months can be more effective than the aerobic training

B. Aerobic exercise, if done without proper training can lead to injuries in neck and back bone.

C. 70 % of the people who reduce fat by aerobic training have gained weight in the next 6 months

D. Aerobic training if not combined with resistive training can reduce the muscle's strength.

E. Research center X's study has not been verified independently. Although, the sample were from diverse communities, the total number of people were not enough to draw a conclusion
"


The answer is A. I'm confused why not E. What if the research center study is flawed.
@arunss90 said:
Hi all, I am bit confused with the CR question I read in a forumThe question is"A recent study has found that aerobic exercising is the best bet to reduce fat compared to resistive training. A research center X had done a head-to-head comparison on aerobic training, resistive training, combo of both and found thataerobic exercising is the most efficient and effective way to reduce one's belly fat. Aerobic exercise reduce 70% more calories compared to resistance training.Which of the following if true, weakens the argument?A. Pilates training, if done for regularly for 6 months can be more effective than the aerobic trainingB. Aerobic exercise, if done without proper training can lead to injuries in neck and back bone.C. 70 % of the people who reduce fat by aerobic training have gained weight in the next 6 monthsD. Aerobic training if not combined with resistive training can reduce the muscle's strength.E. Research center X's study has not been verified independently. Although, the sample were from diverse communities, the total number of people were not enough to draw a conclusion"The answer is A. I'm confused why not E. What if the research center study is flawed.
aerobic training ( cause ) ---- > reduce fat ( effect ) : in addition to this, there is a comparative degree of effectiveness, denoted by "best" --- so no other cause (=training/exercise) is more effective than aerobic training.

A - mentions another cause, more effective than aerobic training over a particular time period. but, this is not a good answer and its not the way causality questions actually works. here, we are assuming that aerobic is "best" as per the claim and this in fact define the scope of the argument, or to extend, we can broaden the scope a little by considering "aerobic training, resistive training or both". So "other training = pilates traning" is actually outside the scope of the argument. An analogy, drinking coffee regularly increases probability for cancer ---- we can't weaken it by --- food taken along with coffee actually increases the probability for cancer. infact "food" is out of scope for this weaken question ( but not for necessary assumption question ).
the other way of thinking is "extended time period = 6 months", which is also very much outside the scope of the argument.

E is bad at the same time - when an argument draws any conclusion from a set of experiments ( sample ), the argument gets weakened if the validity of the study is proved to be false. But, E do not do that --- E present us with a another claim without justification - "the total number of people were not enough to draw a conclusion" : why ??? moreover , GMAT/LSAT answer choices are not written in this format.

so, ignore this !!

Thanks @avik.ch. In your explanation, yo said like the argument gets weakened if the validity is proved false. Isnt wat they did in choice E?. they say like the X's research is not yet verified.

@arunss90 said:
Thanks @avik.ch. In your explanation, yo said like the argument gets weakened if the validity is proved false. Isnt wat they did in choice E?. they say like the X's research is not yet verified.
a claim cannot not weaken or strengthen any argument, because a claim by itself cannot prove anything.
@avik.ch Then what could be the possible answer choice for the question?

@avik.ch i think the answer shd be "E" cause here the argument scope is limited with aerobic and resistive training. the claim is of aerobic training being best bet to reduce fat compared to resistive training and not compared to all the trainings there in the world for that matter. by that standard option A is out of scope...option E is well within the scope and tries to negate the argument by showing that the data used is inappropriate/insufficient to draw conclusion.

As far as option A and E are compared, both are claims without justification.

arunss90Qn
Here is another CR question. Can someone please help me find the answer?

Qn : Any film produced by SONY entertainment would gross at least $100m. Also known is that any movie made for less than $20m will not gross more than $150m. A film that sees a guest appearance from Brad pitt will gross no less than $120m

If all of the above are true, which of the following must be true?

A. A film that has a guest appearance from Brad pitt cannot be made for less than $20m

B. Sony entertainment cannot produce a movie that costs less than $20m

C. It is possible that a movie with a guest appearance from Brad pitt is profuced by Sony entertainment at a cost of less than $20m

D. A Brad pitt starrer will gross more than a movie produced by Sony entertainment.

E. A sony entertainment movie with Brad pitt in a guest appearance will definitely gross more than the movie made for less than $20m


Answer please

@arunss90 its C.
@blitz83 said:
@avik.ch i think the answer shd be "E" cause here the argument scope is limited with aerobic and resistive training. the claim is of aerobic training being best bet to reduce fat compared to resistive training and not compared to all the trainings there in the world for that matter. by that standard option A is out of scope...option E is well within the scope and tries to negate the argument by showing that the data used is inappropriate/insufficient to draw conclusion.As far as option A and E are compared, both are claims without justification.
A is not a claim, instead its an evidence.

E is indeed a claim - E would be correct if the answer choice would be framed in this way -

1- the people included in the survey also undergone a strict diet schedule during the period of experiment -- or,
2 - the sample included people who constituted lower calorie intake than the national average.

something, like this would be correct .... here both 1 and 2 are evidence and not claims. Moreover, no GMAT/LSAT answers are framed in this format as E. None of the answer choice is an apt weakener, so I doubt the source of this question.

Yes the answer is C for the sony entertainment question..I was confused with E adding up 100 and 120..but thats not the case always. so C is the answer πŸ˜ƒ

I found the proper explanation for the aerobic training exercise question from the same site i read picked the question., E is incorrect because it questions the premise ( credibility of the X's research study). Never question the premise.

@arunss90 said:
Never question the premise.
you can definitely question the premise, if the premise is a claim,assertion of facts, result of a survey, .....

when premise is a fact or an evidence, then your argument is valid, but that is not the case here.

@avik.ch yo can only question the conclusion derived from a premise and yo can never question a premise in GMAT even if we know that the premise has a flaw.. Premises are undisputed facts that can never be questioned.
@arunss90 said:
@avik.ch yo can only question the conclusion derived from a premise and yo can never question a premise in GMAT even if we know that the premise has a flaw.. Premises are undisputed facts that can never be questioned.
what is the source of this information --

anyway, refer this OG question :

The difficulty with the proposed high-speed train line is that a used plane can be bought for one-third the price of the train line, and the plane, which is just as fast, can fly anywhere. The train would be a fixed linear system, and we live in a world that is spreading out in all directions and in which consumers choose the free-wheel systems (cars, buses, aircraft), which do not have fixed routes. Thus a sufficient market for the train will not exist.

Which of the following, if true, most severely weakens the argument presented above?

(a) Cars, buses, and planes require the efforts of drivers and pilots to guide them, whereas the train will be guided mechanically.
(b) Cars and buses are not nearly as fast as the high-speed train will be.
(c) Planes are not a free-wheel system because they can fly only between airports, which are less convenient for consumers than the high-speed train €ℒs stations would be.
(d) The high-speed train line cannot use currently underutilized train stations in large cities.
(e) For long trips, most people prefer to fly rather than to take ground-level transportation.


here the correct answer C -- actually weakens the premise, which is a claim.

there are numerous LSAT and GMAT question that deals with premise, where the premise is not an evidence.

@avik.ch your knowledge is badly needed for this question...

Although fullerenes- spherical molecules made entirely of carbon-were first found in laboratory , they have since have been found in nature, formed in fissures of the rare mineral Shungites. since laboratory synthesis of fullerenes requires distinctive conditions of temperature and pressure, this discovery should give geologists a test case for evaluating hypotheses about the state of the earth's crust at the time these naturally occuring fullerenes were formed.

Which of the following ,if true, most seriously undermines the argument?

1. confirming that the Shungite genuinely contained fullerenes took careful experimentation.

2. Some fullerenes have also been found in the remains of a meteorite that collided with a spacecraft.

3. The mineral shungite itself contains large amounts of carbon, from which the fullerenes apparantly formed.

4. The naturally occuring fullerenes are arranged in a previously unknown crystalline structure.

5. Shungite itself is formed under distinctive conditions.

Please explain...thanks in advance.


 

1. Advertisement: Today's clients expect top quality. Every improvement in the standards of service raises client
expectations. The company that is complacent and satisfied with the current quality of its products will soon find
that its clients are not. At Service Enterprises, meeting or exceeding client expectations is our goal.
Which of the following must be true on the basis of the statements in the advertisement above?
A. Service Enterprises' competitors will succeed in attracting customers only if those competitors adopt Service
Enterprises' goal as their own.
B. A company that does not correctly anticipate the expectations of its customers is certain to fail in advancing
the quality of its products.
C. Service Enterprises goal is possible to meet only if continuing advances in product quality are possible.
D. If a company becomes satisfied with the quality of its products, then the quality of its products is sure to
decline.
E. Service Enterprises' customers are currently satisfied with the quality of its products


I feel its C! i didnt find the answer to this when this was posted earlier!

@blitz83 is OA - 3?